Verwirrung über Rotationen von Quantenzuständen: SO(3)SO(3)SO(3) versus SU(2)SU(2)SU(2)

Ich versuche, die Beziehung zwischen Rotationen im "realen Raum" und im Quantenzustandsraum zu verstehen. Lassen Sie es mich an diesem Beispiel erklären:

Angenommen, ich habe ein Spin-1/2-Teilchen, sagen wir ein Elektron, dessen Spin im gemessen wird z + Richtung. Wenn ich dieses Elektron um einen Winkel von drehe π um den Spin in die zu bekommen z Richtung "dreht" sich der Quantenzustand um den halben Winkel ( π / 2 ) wegen der Orthogonalität der Zustände | z + Und | z . Ich denke, das ist nicht sehr streng, aber ist diese Sichtweise richtig?

Ich suchte danach, wie ich dieses Ergebnis herleiten könnte, und fing an, etwas über Repräsentationen zu lernen. ich lese über S Ö ( 3 ) Und S U ( 2 ) und ihre Beziehung, aber es ist mir immer noch unklar. Ich fand diese Aktion von S U ( 2 ) auf Spinoren:

Ω ( θ , N ^ ) = e ich θ 2 ( N ^ σ ) ,
Wo N ^ ist eine Einheit 3 D Vektor, σ = ( σ 1 , σ 2 , σ 3 ) ist der Pauli-Vektor und σ ich sind die Pauli-Matrizen. Ich sehe den Faktor von 1 2 auf den Drehwinkel θ , aber wo kommt das her?

ich sah [ σ ich , σ J ] = 2 ich ϵ ich J k σ k , und machen X J = ich 2 σ J der Kommutator wird [ X ich , X J ] = ϵ ich J k X k , das ist der Kommutator der S Ö ( 3 ) Lügen-Algebra, nicht wahr? Wenn ich also die Exponentialzahl berechne

e θ ( N ^ X ) = e ich θ 2 ( N ^ σ ) .
Ich bekomme mein Ergebnis und es scheint, als wäre es eine Drehung, aber ich habe gelesen, dass es keine ist S Ö ( 3 ) Darstellung. Wo erscheinen Rotationen?

Meine zentrale Frage ist jedoch: Wie kann ich zeigen, dass eine Rotation auf "unserer Welt" eine Rotation von Quantenzuständen erzeugt, und wie verwende ich dies, um die Formel für Rotationen auf Quantenzuständen zu zeigen? Und wie mache ich das für höhere Spinwerte? Ich bin wirklich neu in diesem Thema und es war schwierig, diese Frage zu formulieren. Fragen Sie mich also nach einer besseren Erklärung oder um Missverständnisse auszuräumen.

Ich denke, ein Teil Ihrer Frage ist, wie eine SU (2) -Wiederholung den Drehimpuls trägt. Dies ergibt sich aus der Anforderung, dass auf alles, was einen Drehimpuls trägt, ein Generator wirkt Ö der den Drehimpuls an die Messgeräte koppelt J im Hamiltonian durch Δ H = J . Ö . Diese Durchschnitts Ö muss der Lie-Algebra von SO(3) gehorchen (obwohl sie nicht global gleich sein muss). SU(2) qualifiziert sich dafür und somit ist der niedrigste Drehimpuls rep bei Casimir 1 2 . Sobald Sie dies haben, folgt der Rest.
Ich habe unten die Frage basierend auf meinem obigen Kommentar beantwortet.

Antworten (3)

  1. Einerseits, wenn a R 3 bezeichnet einen 3D-Rotationsvektor 1 , dann die entsprechende Rotationsmatrix

    (1) R ( a )   =   exp ( ich a L )     S Ö ( 3 )     M A T 3 × 3 ( R ) .
    Hier ich L J S Ö ( 3 ) M A T 3 × 3 ( R ) sind drei S Ö ( 3 ) Lügenalgebra-Generatoren, definiert als
    ich ( L J ) k   =   ϵ J k , J , k ,     { 1 , 2 , 3 } , (1') ϵ 123   =   1 ,
    und Erfüllung der S Ö ( 3 ) Lie-Klammer-Beziehungen
    (1") [ L J , L k ]   =   ich = 1 3 ϵ J k L , J , k ,     { 1 , 2 , 3 } .

  2. Andererseits die entsprechende S U ( 2 ) Matrix ist 2

    (2) X ( a )   =   exp ( ich 2 a σ )     S U ( 2 )     M A T 2 × 2 ( C ) ,
    Wo σ sind die Pauli-Matrizen .

  3. Die beiden Matrixdarstellungen (1) & (2) sind über miteinander verbunden

    (3) X ( a ) σ k X ( a ) 1   =   J = 1 3 σ J R ( a ) J k , R ( a ) J k   =   1 2 T R ( σ J X ( a ) σ k X ( a ) 1 ) , J , k     { 1 , 2 , 3 } .

  4. Die Beziehung (3) legt die Tatsache offen, dass die adjungierte Darstellung

    (4) A D :   S U ( 2 ) S Ö ( S u ( 2 ) )     S Ö ( 3 ) ,
    gegeben von
    (4') X A D ( X ) σ   :=   X σ X 1 , X     S U ( 2 ) , σ     S u ( 2 )   :=   { σ M A T 2 × 2 ( C ) σ = σ     T R ( σ ) = 0 }   =   S P A N R { σ 1 , σ 2 , σ 3 } , | | σ | | 2   =   det ( σ ) , A D ( ± 1 2 × 2 )   =   1 S u ( 2 ) ,
    ist ein surjektiver 2:1 -Lie-Gruppenhomomorphismus zwischen S U ( 2 ) Und S Ö ( 3 ) , dh
    (4") S U ( 2 ) / Z 2     S Ö ( 3 ) .

  5. Siehe auch diesen verwandten Phys.SE-Beitrag, der das Halbwinkel-Erscheinungsbild in Gl. (2).

Verweise:

  1. G. 't Hooft, Introduction to Lie Groups in Physics , Vorlesungsunterlagen, Kapitel 3 & 6. Die PDF-Datei ist hier verfügbar .

--

1 Die Richtung eines Rotationsvektors a R 3 ist parallel zur Rotationsachse, während die Länge | a | bezeichnet den Rotationswinkel gegen den Uhrzeigersinn (von der Spitze des Rotationsvektors aus gesehen).

2 Die Notation und Konventionen in dieser Phys.SE-Antwort folgen Ref. 1.

Obwohl an Ihrer Antwort technisch nichts auszusetzen ist, habe ich das Gefühl, dass sie die Frage physisch nicht wirklich beantwortet. Es behauptet im Grunde nur, dass sich der Drehimpulsoperator in die adjungierte Darstellung umwandelt. Meiner Meinung nach geht es insbesondere nicht darauf ein, dass das OP die „zentrale Frage“ aufruft: Wie kann ich zeigen, dass eine Rotation auf „unserer Welt“ eine Rotation von Quantenzuständen erzeugt, und wie verwende ich das, um dies zu zeigen? Formel für Drehungen auf Quantenzuständen?
Ich habe die Antwort aktualisiert.

Das ist eine sehr gute Frage. Es gibt mehrere Möglichkeiten, Ihre Frage zu beantworten. Ich erinnere mich, dass ich die ganze Gruppentheorie verstanden hatte und dennoch das Gefühl hatte, die Physik nicht verstanden zu haben, als ich zum ersten Mal auf das Problem stieß, also werde ich versuchen, die Dinge physikalisch zu erklären.

Zuerst sollte man sich fragen, woher man weiß, dass etwas einen Drehimpuls hat und ihn misst? Ich spreche nicht nur von Spin, sondern auch von Alltagsgegenständen. Sie würden es mit etwas anderem koppeln, das den Drehimpuls aufnehmen kann. Wie ein Baseballschläger.

Im Falle von Elektronen kommt es von der Spin-Bahn-Kopplung oder Emission von Photonen, die Spin tragen. Wir haben jedoch ein viel zugänglicheres Beispiel, weil Elektronen Ladung tragen. Die Ladung hilft uns, da sich der Drehimpuls dann als magnetisches Moment manifestiert, das an ein Magnetfeld koppelt, und wir können den Stern-Gerlach-Aufbau verwenden, um die Bahnen von Teilchen mit unterschiedlichem Drehimpuls zu beeinflussen (solange sie unter dem üblichen geladen sind U(1) EM-Ladung).

Also im Grunde hat der Hamiltonian eine Kopplung

Δ H = G   N ^ . J

wo im Fall Stern Gerlach die N ^ entspricht der Magnetfeldrichtung. Invarianz unter infinitesimalen Drehungen bedeutet die J müssen die Algebra erfüllen (Sie sollten versuchen, dies zu beweisen oder eine separate Frage stellen)

[ J ich , J J ] = ϵ ich J k J k

wobei die Summationskonvention für wiederholte Indizes verwendet wird. Die kleinsten Dimensionsoperatoren, die es erfüllen, sind 2 × 2 (ein Ergebnis, das auch als das kleinste Spin-Wesen bekannt ist 1 2 ) und diese Operatoren werden von der Basis überspannt { 1 2 σ X , } .

Beachten Sie an dieser Stelle eine mathematische Tatsache, die ich nie zu beweisen versucht habe: Bei halbintegralen Drehimpulsoperatoren deckt die Potenzierung die Rotationsgruppe nicht ab S Ö ( 3 ) sondern deckt S U ( 2 ) . Diese sind lokal isomorph und das ist alles, was von der Kopplung im Hamilton-Operator verlangt wird. Mit anderen Worten Darstellungen von S U ( 2 ) kann mit klassischen Instrumenten koppeln, die den Drehimpuls messen (bedeutet nicht, dass sie zum Beispiel SU (2) Teilmenge der Farbe SU (3) nicht müssen).

Also schließen wir das für den Spin 1 2 der allgemeinste Drehimpulsoperator ist

Ö = 1 2 N ^ . σ

Wo N ^ ist ein Einheitsvektor in irgendeiner Richtung.

Jetzt können wir eine Menge Mathematik und Gruppen-/Darstellungstheorie machen, um zu sagen, dass sich dies in die adjungierte Darstellung umwandelt, oder wir können die Physik sehen. Angenommen, wir führen eine Winkeldrehung durch θ um die durch den Einheitsvektor gegebene Achse M ^ . Was erwarten wir? Wir würden erwarten, einen neuen Operator zu bekommen, der dem Rotieren entspricht N ^ von θ um M ^ . Ein solcher Vektor ist

N ^ ~ = ( N ^ . M ^ ) M ^ + ( N ^ ( N ^ . M ^ ) M ^ ) cos θ + ( M ^ × N ^ ) Sünde θ

und der gedrehte Operator ist

Ö ~ = 1 2 N ^ ~ . σ

Nun ist die Behauptung, dass der Betreiber Ö transformiert in den adjungierten rep und das bedeutet, dass wir bekommen sollten

Ö ~ = 1 2 e ich θ 2 M ^ . σ   N ^ . σ   e ich θ 2 M ^ . σ

Das tut es tatsächlich und der Vollständigkeit halber werde ich es hier herleiten. Wir brauchen

( A . σ ) ( B . σ ) = ( A . B ) + ich ( A × B ) . σ

Wir sehen das

1 2 e ich θ 2 M ^ . σ   N ^ . σ   e ich θ 2 M ^ . σ = [ cos ( θ 2 ) ich ( M ^ . σ ) Sünde ( θ 2 ) ] [ N ^ . σ ] [ cos ( θ 2 ) + ich ( M ^ . σ ) Sünde ( θ 2 ) ] = [ cos ( θ 2 ) ich ( M ^ . σ ) Sünde ( θ 2 ) ] [ ( N ^ . σ ) cos ( θ 2 ) + ich ( N ^ . M ^ + ich ( N ^ × M ^ ) . σ ) Sünde ( θ 2 ) ] = ( N ^ . σ ) cos 2 ( θ 2 ) + 2 ( M ^ × N ^ ) Sünde ( θ 2 ) cos ( θ 2 ) + ( N ^ . M ^ ) M ^ . σ Sünde 2 ( θ 2 )       ( N ^ ( N ^ . M ^ ) M ^ ) . σ Sünde 2 ( θ 2 ) = N ^ ~ . σ

Wir sehen also, dass sich der Operator tatsächlich in die adjungierte Darstellung transformiert.

Nun der Erwartungswert für einen Zustand | ψ Ist

ψ | Ö | ψ

und daher bedeutet Invarianz unter Rotationen das

| ψ ~ = e ich θ 2 M ^ . σ | ψ = [ cos ( θ 2 ) ich ( M ^ . σ ) Sünde ( θ 2 ) ] | ψ

was zeigt, dass unter einer Drehung durch π wir bekommen | ψ ~ = ich ( M ^ . σ ) | ψ wie der OP gefragt hat.

Die Verallgemeinerung auf höhere Spins ist nicht einfach, da wir keine Eigenschaften wie haben J X 2 = 1 und man muss die Baker-Campbell-Hausdorff-Formel verwenden. Sobald die obige Idee klar ist, kann man jedoch die Ergebnisse der Gruppentheorie verwenden, um zu sehen, dass wir zum Beispiel für Spin-1 3 haben werden 3 × 3 Matrizen

J X = 1 2 ( 0 1 0 1 0 1 0 1 0 ) J j = 1 2 ( 0 ich 0 ich 0 ich 0 ich 0 ) J z = ( 1 0 0 0 0 0 0 0 1 )

und wieder wird der allgemeinste Operator sein

Ö 3 = N ^ . J

Da der Operator nun aus den Generatoren von SO(3) besteht, wissen wir, dass er sich unter dem adjungierten rep transformieren wird, und obwohl es erheblich mehr Arbeit als oben erfordern wird, um zu beweisen, dass dies der Fall ist, ist die Bedeutung einer solchen Transformation dies

Ö 3 ~ = N ^ ~ . J

und dann wird uns das gleiche Argument wie oben liefern, dass sich der Zustand in die fundamentale Darstellung als umwandelt

| ψ 3 ~ = e ich θ   N ^ . J | ψ 3

Alternativ stellt man das Spin-1-Teilchen als Triplettzustand zweier Spin- 1 2 Partikel und arbeitet von dort.

Ok, das mit den anderen Antworten klärt mich sehr auf. Aber ich habe immer noch Zweifel an diesem Ausdruck Ö = 1 2 N ^ . σ Ich kann nicht sehen, woher das kommt 1 2 ist es für die Erfüllung der Algebra von J ? Multiplizieren mit einer Zahl ergibt nicht die gleiche Algebra?
Eine Beziehung mit einem Produkt von 2 Operatoren auf der linken Seite und 1 auf der rechten Seite ist unter Skalierung dieser Operatoren nicht unveränderlich. Versuch es. Die SU(2)-Algebra wird von Pauli-Matrizen nicht befolgt, sondern nur halbiert.
Okay, das war das Puzzleteil, das mir gefehlt hat. Ich werde versuchen, es selbst und das andere, was Sie in Ihrer Antwort erwähnt haben, zu beweisen. Danke schön!

Wie kann ich zeigen, dass eine Drehung auf „unserer Welt“ eine Drehung von Quantenzuständen erzeugt, und wie verwende ich das, um die Formel für Drehungen auf Quantenzuständen zu zeigen?

Wie bei allem in der Physik stellt man Postulate auf und probiert sie aus. Die Postulate, die Sie aufstellen müssen, sind jedoch sehr, sehr mild und offensichtlich, und diese ermöglichen es Ihnen dann, Wigners Theorem zu verwenden , das ein mächtiges Tier ist, das die Dinge wirklich genau für die Frage festnagelt, die Sie oben stellen.

Wenn wir unser Quantenobjekt oder unser Koordinatensystem drehen, stellen wir das sehr offensichtliche Postulat auf, dass diese Transformation, welche Transformation auch immer diese Aktion im Quantenzustandsraum bewirkt, die inneren Produkte im Quantenzustandsraum bewahren muss, damit der Zustand erhalten bleibt richtig normalisiert.

Allein aus diesen Postulaten, dh man muss nicht einmal von Linearität ausgehen , hat Wigner bewiesen, dass das, wenn unser Quantenobjekt/Koordinatensystem mehrere aufeinanderfolgende "Symmetrien" ( also Lorentz-Transformationen, die natürlich Rotationen beinhalten) erfährt, das Entsprechende Quantenzustandsraumtransformationen müssen "kompatibel mit der Symmetriekomposition komponieren" (der Fachausdruck, den man in diesem Zusammenhang hört). Dies wird in Symbolen genauer ausgedrückt. Lassen σ : S Ö ( 1 , 3 ) T sei die Abbildung zwischen den "Symmetrien" (Lorentz-Transformationen, einschließlich Drehungen in S Ö ( 1 , 3 ) und die Menge der Quantenzustandsraumtransformationen T . Dann haben wir:


Wigners Theorem

Angesichts der obigen Postulate (siehe an anderer Stelle für eine genauere Aussage) für zwei beliebige γ , ζ S Ö ( 1 , 3 ) , die entsprechenden Quantenzustandsraumtransformationen σ ( γ ) Und σ ( ζ ) :

  1. wirken linear oder antilinear auf den Quantenzustandsraum (und sind offensichtlich unitär / antiunitär, da sie innere Produkte erhalten müssen); Und
  2. σ ( γ ζ ) = ± σ ( γ ) σ ( ζ )

dh die Zuordnung σ ist ein sogenannter projektiver Homomorphismus .

Der ± Zeichen in der obigen Gleichung ist belanglos und gewichtig zugleich! Es ist insofern belanglos, als Quantenzustände eigentlich Strahlen im Quantenzustandsraum sind, das Vorzeichen (wie jede globale Phase) keinen Einfluss auf die physikalische Wirkung der Transformation auf den Zustandsraum hat. Aber die Tatsache, dass es dort dieses Zeichen gibt, bedeutet Folgendes:

  1. Wir haben es nicht nur mit Repräsentationen der Gruppe zu tun S Ö ( 1 , 3 ) (oder S Ö ( 3 ) im Kontext Ihrer Frage) (was der Fall des "+" -Zeichens ist); ABER
  2. Es könnte sich um die Darstellung eines topologischen Überlagerungsraumes der Gruppe handeln S Ö ( 1 , 3 ) oder S Ö ( 3 ) (in diesem Fall unsere σ kann die hinzufügen Phase).

Die Idee der topologischen Abdeckung des Raums ist hier entscheidend. Für S Ö ( 1 , 3 ) Und S Ö ( 3 ) es gibt jeweils nur zwei mögliche Deckungen: die Gruppen selbst und ihre universellen Deckungen S L ( 2 , C ) Und S U ( 2 ) , bzw. Mehr gibt es nicht, weil beides S L ( 2 , C ) Und S U ( 2 ) sind einfach zusammenhängend, und ein einfach zusammenhängender topologischer Raum lässt keine nicht-trivialen Überdeckungsräume zu. Es gibt einen lesbaren Beweis dafür in dem alten Buch von WS Massey, "Algebraic Topology: An Introduction".

Die entscheidende Beziehung ist also die S U ( 2 ) ist die doppelte (und universelle, daher die einzige nicht triviale) Deckung von S Ö ( 3 ) .

Jetzt geben Ihnen sowohl die Antwort von Quantum Mechanic als auch Borun Chowdhurys mehr physikalische Antwort ziemlich viele Details der Berechnungen S U ( 2 ) Und S Ö ( 3 ) und ihrer Darstellungen, aber wenn Sie versuchen möchten, ihre Beziehung zu visualisieren, dann werfen Sie vielleicht einen Blick auf die zweite Hälfte dieser Antwort von mir , wo ich versuche, die Standardkonstruktion der angewandten topologischen universellen Überdeckungsgruppe zu veranschaulichen S Ö ( 3 ) damit du es visualisieren kannst S U ( 2 ) als bestehend aus zwei Kopien von S Ö ( 3 ) .